Derivation of Marshallian Demand Functions from Utility Function

Поделиться
HTML-код
  • Опубликовано: 22 авг 2024
  • Derivation of Marshallian Demand Functions from Utility Function
    Learn how to derive a demand function form a consumer's utility function. In this problem, U = 2X(1)^0.5 + 4X(2)^0.5.
    This video introduces the Cobb-Douglas utility function for two goods and demonstrates some of its properties.
    I demonstrate how to compute marginal rate of substitution for Cobb Douglas utility. I use the resulting expression to show why Cobb Douglas indifference curves have constant slope along rays out of the origin.
    I also demonstrate how to solve for consumer demand using the Cobb-Douglas utility function. After deriving Cobb-Douglas demand, I show that the alpha share parameter in Cobb-Douglas utility functions actually represents the consumer's expenditure share on that good.
    This video uses calculus to compute MRS, but after that, the rest is just algebra. If you are a viewer who doesn't know calculus, you can take me at my word, skip the computation of MRS, and the rest of the video can still be worthwhile.
    Economics Playlist
    Derivation of Marshallian Demand Functions from Utility Functions: • Derivation of Marshall...
    Derivation of Marshallian Demand Functions from Utility Functions (II): • Derivation of Marshall...
    Derivation of Hicksian Demand Functions from Utility Functions:
    • Derivation of Hicksian...
    Deriving Marshallian and Hicksian Demand Functions (compensated and uncompensated demand):
    • Deriving Marshallian a...
    Derivation of a Generalised (n-good) Marshallian Demand Function from Utility Function.
    • Derivation of a Genera...
    Deriving Hicksian (compensated) Demand Using the Slutsky Equation:
    • Deriving Hicksian (com...
    Deriving Marshallian Demand From a Stone-Geary Utility Function:
    • Deriving Marshallian D...

Комментарии • 108

  • @HaroldWalden
    @HaroldWalden  7 лет назад +37

    If you would like to see more of these more advanced microeconomics exam questions, let me know.
    :)

    • @AntwainDCurry
      @AntwainDCurry 6 лет назад +2

      As an addendum to this video please answer the question, "Does the following demand functions satisfy WARP?" by constructing the Slutsky matrix.
      Solution: We know that WARP implies CLD, which in turn implies that S is negative semidefinite.
      Thus, if we show that S is not n.s.d. then WARP is notsatisfied.
      s11 = ∂x1/∂p1 + ∂x1/∂w x1 = p2/(p1 + p2)2 + p1/p2(p1 + p2) > 0,
      which implies that S is not n.s.d. and WARP is not satisfied.
      But, I want to see it worked out.

    • @abdiazizahmed9553
      @abdiazizahmed9553 6 лет назад +4

      Please make a video on deriving slutsky equation through maximization of utility function subject to budget constraint

    • @TheDavePerrin
      @TheDavePerrin 6 лет назад +2

      Most definitely, I'm in microeconomic theory right now at uni and this was extremely helpful. Also if you have anything for macroeconomic theory a few videos on that would be killer as well.

    • @adebayojjulius6056
      @adebayojjulius6056 4 года назад +1

      Plz let's have more lectures and questions as well. Thanks

  • @user-hj4xf5fu8w
    @user-hj4xf5fu8w 3 года назад +6

    wow, you made this way less complicated than what it seemed in my intermediate micro book. thanks a lot for taking the time to make this!!

  • @huseyinalupveren3768
    @huseyinalupveren3768 2 года назад +2

    one of the best micro videos ive ever watched. but i recommend to use different letters instead using x1 and x2.

    • @HaroldWalden
      @HaroldWalden  2 года назад +3

      Good call! It also gets pretty damn confusing and it’s a accident waiting to happen algebraically 😑

  • @MathsHistoryHelp
    @MathsHistoryHelp 4 года назад +10

    God bless you sir. I'm trying to get through Hal Varians advanced microeconomics and it ain't pretty, but this helps a great deal.

  • @GMan-um1pi
    @GMan-um1pi 6 лет назад +5

    Explained better than a UC Berkeley professor - thanks!

    • @HaroldWalden
      @HaroldWalden  6 лет назад +3

      +Grady haha, thanks. Just trying to help some folks out.
      The professor that explained this to me was also hopeless. It took one of my mates from the year above to start from scratch and explain it without the jargon for the penny to drop with me.
      I am glad that it helped :)

    • @carlawalden
      @carlawalden 6 лет назад +3

      That’s exactly what I said. Kid needs more views.

  • @keatonfitzpatrick8210
    @keatonfitzpatrick8210 4 года назад +2

    Thank you very much Harold. I have just completed a similar question that I received as an assignment

    • @HaroldWalden
      @HaroldWalden  4 года назад +1

      Good to hear that my videos are helpful . Thanks for taking the time to leave a positive comment 🤓

  • @joejayray
    @joejayray 5 лет назад +9

    I think there is an error in setting up the Lagrangian. U(x)+lambda(Y-P1X1-P2X2) or perhaps U(x)-lambda(P1X1+P2X2-Y).
    Because the way it has been solved in the video, the FOCs are wrong.

    • @HaroldWalden
      @HaroldWalden  5 лет назад +2

      Can you identify exactly where the mistake is, I don’t follow what you mean.

    • @HaroldWalden
      @HaroldWalden  5 лет назад +3

      Bit of an update; the sign of the Lagrange multiplier would affect the value of the first order derivative however as long as you are consistent with your signs throughout the problem you shouldn't get a different answer.
      I hope that helps :)
      Here is a link to a great explanation on this exact topic:
      math.stackexchange.com/questions/1099429/when-using-the-method-of-lagrangian-multipliers-does-it-matter-whether-i-subtra

  • @wahing999
    @wahing999 5 лет назад +12

    ins't the Lagrangian equation + lambda, not - lambda?

    • @HaroldWalden
      @HaroldWalden  5 лет назад +4

      in these cases I don't think it matters.

    • @HaroldWalden
      @HaroldWalden  5 лет назад +6

      Bit of an update; the sign of the Lagrange multiplier would effect the value of the first order derivative however as long as you are consistant with your signs throughout the problem you shouldn't get a different answer.
      I hope that helps :)
      Here is a link to a great explanation on this exact topic:
      math.stackexchange.com/questions/1099429/when-using-the-method-of-lagrangian-multipliers-does-it-matter-whether-i-subtra

  • @yuhaoyang2830
    @yuhaoyang2830 4 года назад +5

    I think the function should be : L = U(x1, x2) + λ (M - PxX - PyY), not - λ.
    Is that correct?

    • @HaroldWalden
      @HaroldWalden  4 года назад +5

      It’s fine either way, the first order conditions will cancel to make the sign on the lambda irrelevant.
      Good question though
      🤓

  • @mattpalucci3855
    @mattpalucci3855 4 года назад +7

    you saved my micro mark thank you lol

    • @HaroldWalden
      @HaroldWalden  4 года назад +2

      That great to hear. I am happy that I could be of assistance :)

  • @danmchardy6424
    @danmchardy6424 6 лет назад +5

    I don't understand why in the original langrangian equation some choose to add the constraint and some choose to minus the constraint. Wouldn't this affect the first-order derivative?

    • @HaroldWalden
      @HaroldWalden  6 лет назад +4

      Hi Dan, the sign of the Lagrange multiplier would effect the value of the first order derivative however as long as you are consistant with your signs throughout the problem you shouldn't get a different answer.
      Here is a link to a great explanation on this exact topic: math.stackexchange.com/questions/1099429/when-using-the-method-of-lagrangian-multipliers-does-it-matter-whether-i-subtra
      Hope it helps :)

  • @adelezito7342
    @adelezito7342 3 года назад +3

    Hello! What can we do if the partial derivative of x2 is equal to 1 ? How are we able to to plug x2 in the Y formula ?

  • @kylehammerberg3875
    @kylehammerberg3875 3 года назад

    Incredibly concise. Thank you.

    • @HaroldWalden
      @HaroldWalden  3 года назад

      Thanks for the compliment, I really appreciate it :)

  • @sports4160
    @sports4160 6 лет назад +33

    This is scary than a horror movie

    • @HaroldWalden
      @HaroldWalden  6 лет назад +3

      Yep, that’s what I thought when my lecturer first went through it.

  • @kuortayi7425
    @kuortayi7425 6 лет назад +2

    Perfectly explain thanks.

    • @HaroldWalden
      @HaroldWalden  6 лет назад +2

      +Gedleyihlekisa Dunah thanks, I am glad it helped. I always struggled to find a good way of explaining how these optimisation questions/problems work. I feel I am getting closer but I am going to keep trying.

  • @itsteej
    @itsteej 2 года назад +1

    I hate math but you make me hate it a bit less but I still hate it a lot.

  • @pamipami6838
    @pamipami6838 5 лет назад +2

    Thanks so much i really enjoy the content can you please show how to get the Hicksian demand after getting the marshallian

    • @HaroldWalden
      @HaroldWalden  5 лет назад +2

      Hey, already have put together a video answering exactly that.
      ruclips.net/video/Zrs9uSMg6Sg/видео.html
      Have a look and if it doesn’t help let me know :)

  • @tarabourhis1282
    @tarabourhis1282 3 года назад +2

    Why is the lambda have a negative in front of it, I thought the formula for the lagrangian was with a positive for maximizing utility.

    • @HaroldWalden
      @HaroldWalden  3 года назад +4

      Try it with either positive or negative, I think you’ll find it doesn’t make a difference 🤓

    • @tarabourhis1282
      @tarabourhis1282 3 года назад +1

      @@HaroldWalden Thank you!

    • @HaroldWalden
      @HaroldWalden  3 года назад +1

      No problem ❤️

  • @boyceeeee1
    @boyceeeee1 6 лет назад +2

    Thank you so much!! Very clear and helpful :)

    • @HaroldWalden
      @HaroldWalden  6 лет назад +2

      Thank's, I am glad that these videos are making life easier for people trying to wrap their heads around these kind of concepts, sometimes it feels like you are just banging your head against a wall. If there are any problems your having, I am always looking for new ideas for videos... Let me know.
      :)

    • @HaroldWalden
      @HaroldWalden  6 лет назад +2

      +Marilou Flandres glad I could help :)
      I think I'll put up more of these slightly more complex economics videos, they seem to be getting a fair bit of love :)

  • @Rundtj45
    @Rundtj45 4 года назад +1

    thank you so much!. It was very helpful.

    • @HaroldWalden
      @HaroldWalden  4 года назад +1

      No problem, glad I could help.
      Thanks for taking the time to leave a positive comment 🤓

  • @roryh2241
    @roryh2241 3 года назад +3

    Hi! I know it's a bit late, but is there any reason why you subtract the Lagrange multiplier? I thought you added it, cheers!

    • @HaroldWalden
      @HaroldWalden  3 года назад +2

      It doesn’t matter, you can add or subtract it and the outcome will be the same.
      Check if you like :)

  • @PaulArgamosa
    @PaulArgamosa 6 лет назад +4

    clear, concise explanation and beautiful video. what software do u use?

    • @HaroldWalden
      @HaroldWalden  6 лет назад +2

      I use a program called Sketchup and a Wacom Bamboo pen tablet and I use a program called Camtasia for my screen capture.

    • @PaulArgamosa
      @PaulArgamosa 6 лет назад +2

      many thanks!

    • @HaroldWalden
      @HaroldWalden  6 лет назад +2

      No worries 🤓

  • @agnesphileoboateng6790
    @agnesphileoboateng6790 5 лет назад +3

    why did you take a reciprocal formula when finding X and Y in the first part

    • @HaroldWalden
      @HaroldWalden  5 лет назад +2

      Which bit exactly are you taking about?

    • @agnesphileoboateng6790
      @agnesphileoboateng6790 5 лет назад +2

      after taking the FOC of x1 and x2 , you have to solve both to obtain x1 and x2 and in your solution you said take the reciprocal of the lamdar P =-2/x2^1/2 . so i'm asking why the reciprocal formula @@HaroldWalden

    • @HaroldWalden
      @HaroldWalden  5 лет назад +2

      @@agnesphileoboateng6790 Oh ok, I understand. So to get LambdaP_1/LambdaP_2 we need to divide -1/(x_2)^(1/2) by -2/(x_2)^(1/2). When dealing with fractions, I find it is easier to use the fact that division is simply multiplication by the reciprocal
      For example, 6 divided by 3 is exactly the same as 6 times by 1/3.
      So taking the reciprocal has nothing to do with the intuition behind the problem, it is simply me trying to verbalize what I find is the easiest way to do the algebra.
      I hope that helps, let me know if I need to clarify it further :)

  • @AamirKhan-og8kv
    @AamirKhan-og8kv 5 лет назад +2

    how can i solve if the function is a + b < 1. that is power does not add up to 1. I am stuck with algebra

    • @HaroldWalden
      @HaroldWalden  5 лет назад +2

      well, in this case, a + b must be less than or equal to one. However, because we are trying to maximise our utility, the only way we can do that is if a + b equal one (if you're using a and b to represent proportions of consumers income spent on goods a and b. In the case you are describing, I would still maintain that assumption to be true and where you have a + b < 1 substitute in a + b = 1. In these type of problems, you should be spending ALL of your income on the two goods as the maximum utility can ONLY be achieved when you are devoting all the resources at your disposal to increase your current utility.
      I hope that helps :)

    • @AamirKhan-og8kv
      @AamirKhan-og8kv 5 лет назад +2

      Thanks that makes sense. I was thinking the same but didn't know i can change the utility function. I have powers of 2/3 and 1/5.
      So i can make it 1/3 instead of 1/5

    • @HaroldWalden
      @HaroldWalden  5 лет назад +2

      Aamir Khan usually the powers will add to one, it might be worthwhile to check with your lecturers to see whether or not there has been a mistake???

  • @ewurafuabentsi-enchill9195
    @ewurafuabentsi-enchill9195 3 года назад

    Please how do u solve for the marshallian demand function with U=xe with e raise to the power y

  • @sulaimanqizilbash4248
    @sulaimanqizilbash4248 6 лет назад +2

    can't we reduce this problem to a single variable problem and solve it?

    • @HaroldWalden
      @HaroldWalden  6 лет назад +2

      Good question, the answer is no. The reason being is because this demand function is based on two price levels that move independently of one another. We can find the demand for each of the goods as a function of the prices of both goods and a fixed income, Y but be cannot simultaneously solve it.

  • @carzzn
    @carzzn 4 года назад +1

    Thank you!

    • @HaroldWalden
      @HaroldWalden  4 года назад +2

      No problem, I hope the video helped 🤓

    • @carzzn
      @carzzn 4 года назад +1

      Harold Walden it absolutely did, taking microeconomic theory and application. Feeling good for class tomorrow :)

    • @HaroldWalden
      @HaroldWalden  4 года назад +2

      Carson yeah it can all seem like gibberish is you haven’t got a handle on this nonsense.

  • @musengebwali5627
    @musengebwali5627 2 года назад

    How do I derive the consumer's Marshallian demand function for min (x,y)

  • @mukasavictor2807
    @mukasavictor2807 3 года назад +1

    Have you stated the lagrangian correctly?

  • @mckenziedallin91
    @mckenziedallin91 5 лет назад +2

    Love the content, but moving the mouse around as much as you do is a bit distracting. Just some feedback!

    • @HaroldWalden
      @HaroldWalden  5 лет назад +2

      Thanks, I am still getting used to the platform. Keeping the mouse still is something I will definitely work on.

  • @mikastupnik
    @mikastupnik 2 года назад +1

    Do you have a video on how to do this but for 3 goods X1, X2 and X3 ?

    • @HaroldWalden
      @HaroldWalden  2 года назад +1

      No but I definitely should do one.

  • @vusalaliyev7990
    @vusalaliyev7990 5 лет назад +2

    So we don't need to find partial derivative of L here :/

    • @HaroldWalden
      @HaroldWalden  5 лет назад +2

      Can you elaborate please?

    • @vusalaliyev7990
      @vusalaliyev7990 5 лет назад +2

      @@HaroldWalden Just got it, thanks. Do you have a video for Stone-Geary utility function, I can't find proper material for this.

    • @HaroldWalden
      @HaroldWalden  5 лет назад +3

      No I don’t but you have definitely given me a great idea for the next video.
      Thanks :)

    • @HaroldWalden
      @HaroldWalden  5 лет назад +3

      Sorry it's a bit late but here is my explanation on Stone-Geary utility functions: ruclips.net/video/qHZHAJozLc8/видео.html
      Hope it helps :)

    • @vusalaliyev7990
      @vusalaliyev7990 5 лет назад +2

      @@HaroldWalden Thank you! That is great!

  • @namratabhoir
    @namratabhoir 3 года назад

    Can you please solve for utility function u=x^1/2+y

  • @viratdubey1966
    @viratdubey1966 3 года назад

    What is y greator and equal

  • @khabophilie7455
    @khabophilie7455 3 года назад +1

    You subtracted the langragian instead of adding

    • @HaroldWalden
      @HaroldWalden  3 года назад +1

      It doesn’t matter if you add or subtract, check if you like.

  • @adebayojjulius6056
    @adebayojjulius6056 4 года назад +1

    Plz sir is this youtube channel still functional???

    • @HaroldWalden
      @HaroldWalden  4 года назад +1

      Yeah mate, I have just been a tad busy :-/

    • @adebayojjulius6056
      @adebayojjulius6056 4 года назад

      @@HaroldWalden thanks so much will always be at alert for lessons

    • @adebayojjulius6056
      @adebayojjulius6056 4 года назад

      Hello, I like to enquire if there is any software that can perform analysis relating to utility or any utility functions software packages. Your reply will be highly appreciated. Thanks

  • @kialyons8761
    @kialyons8761 5 лет назад +1

    I have something similar to what I emailed you. Please let me know if you are interested.

  • @seung-hwankim4410
    @seung-hwankim4410 8 месяцев назад

    You complicate things to much. Useless?

    • @HaroldWalden
      @HaroldWalden  8 месяцев назад

      How would you solve this problem?